Solve Vector Equation Problem: A, B, C & AxB + AxC

  • Thread starter Thread starter jean014
  • Start date Start date
  • Tags Tags
    Vector
Click For Summary
The discussion revolves around solving a vector equation involving an unknown vector A and known vectors B and C. The equation given is (AxB) + (AxC) = (5a + b)i + (3b - 2)j + (-4c + 1)k, where A is expressed as A = ai + bj + cK. Participants suggest starting by calculating A cross B and A cross C, then adding the results to form a system of equations for a, b, and c. The conversation also notes the importance of not posting the same question in multiple threads. The focus remains on providing guidance for solving the vector equation effectively.
jean014
Messages
2
Reaction score
0
hello!

im new here and i just need a little help in solving this problem:
its a set of vector equation:

A = ai + bj + cK
B = -2i +j - 4K
C = i + 3j +2k

where A is an unknown vector. If

(AxB)+(AxC) = (5a + b)i + (3b-2)j + (-4c+1)k

i need to solve for a,b,c..

pls pls pls.. help! >.<

thanks
 
Physics news on Phys.org
solving vector eqns using matrix inversion

hello!

im new here and i just need a little help in solving this problem:
its a set of vector equation:

Homework Statement



A = ai + bj + cK
B = -2i +j - 4K
C = i + 3j +2k

where A is an unknown vector. If

(AxB)+(AxC) = (5a + b)i + (3b-2)j + (-4c+1)k

i need to solve for a,b,c..

Homework Equations



matrix inversion by co-factor mtd or LU decomp..

The Attempt at a Solution



i don't even know where to begin!

pls pls pls help..

just on how to start .. or the algo ill finish the rest >.<

thnx!
 
To start you should calculate A\times(B+C). You know B and C and have an expression for A.
 
jean014 said:
hello!

im new here and i just need a little help in solving this problem:
its a set of vector equation:

A = ai + bj + cK
B = -2i +j - 4K
C = i + 3j +2k

where A is an unknown vector. If

(AxB)+(AxC) = (5a + b)i + (3b-2)j + (-4c+1)k

i need to solve for a,b,c..

pls pls pls.. help! >.<

thanks
Looks a lot like homework to me- so I'm going to move it to the "Homework- calculus and beyond" folder.

And, of course, you will have to make an effort yourself: What are AxB and AxC? That should be your first step. Then add those two and set them equal to the right hand side above. That will give you three equations to solve for a, b, c.
 
This is exactly the same problem as https://www.physicsforums.com/showthread.php?t=188267. They should be merged. Jean, you are not supposed to post the same thing in multiple threads.
 
D H said:
This is exactly the same problem as https://www.physicsforums.com/showthread.php?t=188267. They should be merged. Jean, you are not supposed to post the same thing in multiple threads.
Thanks. I will.

(It was in "Engineering, Computer Science, and Technology"?? No wonder I didn't find it!)
 
Last edited by a moderator:
Question: A clock's minute hand has length 4 and its hour hand has length 3. What is the distance between the tips at the moment when it is increasing most rapidly?(Putnam Exam Question) Answer: Making assumption that both the hands moves at constant angular velocities, the answer is ## \sqrt{7} .## But don't you think this assumption is somewhat doubtful and wrong?

Similar threads

  • · Replies 5 ·
Replies
5
Views
16K
Replies
11
Views
4K
  • · Replies 4 ·
Replies
4
Views
3K
  • · Replies 4 ·
Replies
4
Views
2K
Replies
18
Views
8K
  • · Replies 10 ·
Replies
10
Views
5K
  • · Replies 1 ·
Replies
1
Views
2K
  • · Replies 8 ·
Replies
8
Views
3K
  • · Replies 5 ·
Replies
5
Views
2K
  • · Replies 2 ·
Replies
2
Views
4K